What is the distribution of the sum of n iid Bernoulli random variables?

  • Thread starter Thread starter Qyzren
  • Start date Start date
  • Tags Tags
    Distribution
Click For Summary
The discussion focuses on finding the distribution of the sum of n independent and identically distributed (iid) Bernoulli random variables defined by whether each variable exceeds its mean μ. The variables Yi are defined as 1 if Xi > μ and 0 otherwise, making each Yi a Bernoulli random variable with success probability p, where p is the probability that Xi ≤ μ. The sum ∑ni=1Yi can thus be modeled as a Binomial distribution, specifically Binomial(n, p). The central limit theorem is mentioned, suggesting that for large n, the distribution of the sum approaches a normal distribution with mean nμ and a variance that needs to be determined. Overall, the key takeaway is that the distribution of the sum of these Bernoulli variables can be analyzed using the properties of the Binomial distribution.
Qyzren
Messages
41
Reaction score
0

Homework Statement


Let X1, X2, ..., Xn be iid random variables with continuous CDF FX and suppose the common mean is E(Xi) = μ. Define random variables Y1, Y2, ..., Yn by
Yi = 1 if Xi > μ; 0 if Xi ≤ μ. Find the distribution of ∑ni=1Yi.

I'm having a hard time figuring out how to begin to find the distribution.

Homework Equations


Possibly Yn = (∑Xi - nμ ) /√n σ ?

The Attempt at a Solution


I'm having a hard time knowing where to begin...
If the question was p instead of μ, then Xi ~ Bernoulli(p) and the sum of n iid Bernoulli(p) is Binomial(n, p).

But since we have μ and not p, I'm also thinking that the central limit theorem tells us the sum of any random variable is always yields a normal distribution, with mean nμ? and variance ...?

Any help to get me in the right direction would be greatly appreciated!
 
Last edited:
Physics news on Phys.org
Define ##p_i=Prob(X_i\le \mu)##. Since the ##X_i## are iid, ##p_i## is the same for all ##i## so we can just write it as ##p##. Then each ##Y_i## is a Bernoulli.
 
  • Like
Likes Qyzren
Question: A clock's minute hand has length 4 and its hour hand has length 3. What is the distance between the tips at the moment when it is increasing most rapidly?(Putnam Exam Question) Answer: Making assumption that both the hands moves at constant angular velocities, the answer is ## \sqrt{7} .## But don't you think this assumption is somewhat doubtful and wrong?

Similar threads

  • · Replies 8 ·
Replies
8
Views
1K
Replies
7
Views
2K
  • · Replies 6 ·
Replies
6
Views
2K
  • · Replies 4 ·
Replies
4
Views
2K
  • · Replies 3 ·
Replies
3
Views
1K
  • · Replies 3 ·
Replies
3
Views
2K
Replies
1
Views
1K
  • · Replies 11 ·
Replies
11
Views
4K
  • · Replies 3 ·
Replies
3
Views
3K
Replies
5
Views
2K